Cardiac quizzlet Flashcards

1
Q

During which interval on the EKG does the aortic valve close?

A

T-wave

How well did you know this?
1
Not at all
2
3
4
5
Perfectly
2
Q

Which one of the following is the best index of preload?

A

Left ventricular end-diastolic volume

How well did you know this?
1
Not at all
2
3
4
5
Perfectly
3
Q

Which one of the following is the best index of afterload?

A

Left ventricular mean systolic pressure

How well did you know this?
1
Not at all
2
3
4
5
Perfectly
4
Q

The highest blood flow per gram of left ventricular myocardium would occur

A

At the beginning of diastole

How well did you know this?
1
Not at all
2
3
4
5
Perfectly
5
Q

During ventricular ejection, the pressure difference smallest in magnitude is between the

A

Left ventricle and aorta

How well did you know this?
1
Not at all
2
3
4
5
Perfectly
6
Q

Which of the following would result from a regurgitant aortic valve in a nonfailing heart?

A

A decrease in diastolic pressure

How well did you know this?
1
Not at all
2
3
4
5
Perfectly
7
Q

Stroke volume is increased by

A

A decrease in venous compliance

How well did you know this?
1
Not at all
2
3
4
5
Perfectly
8
Q

An increased preload would most likely be caused by an increase in

A

Venous tone

How well did you know this?
1
Not at all
2
3
4
5
Perfectly
9
Q

Propagation of the action potential through the heart is fastest in the

A

Purkinje fibers

How well did you know this?
1
Not at all
2
3
4
5
Perfectly
10
Q

A patient in heart failure improves markedly after using a drug that increases the inotropic state of her heart. Which one of the following changes is primarily responsible for the improvement in her condition?

A

A reduction in heart size

How well did you know this?
1
Not at all
2
3
4
5
Perfectly
11
Q

Closure of the aortic valve occurs at the onset of which phase of the cardiac cycle?

A

Isovolumetric relaxation

How well did you know this?
1
Not at all
2
3
4
5
Perfectly
12
Q

Which one of the following is most likely to be observed in a patient with untreated atrial fibrillation?

A

An increased left atrial pressure

How well did you know this?
1
Not at all
2
3
4
5
Perfectly
13
Q

Normal splitting of the second heart sound (S2) into two components is increased during inspiration because

A

The closing of the pulmonic valve is delayed

How well did you know this?
1
Not at all
2
3
4
5
Perfectly
14
Q

Which of the following statements about the third heart sound (S3) is correct?

A

It is produced by turbulence during rapid ventricular filling in early diastole

How well did you know this?
1
Not at all
2
3
4
5
Perfectly
15
Q

In a resting, healthy man, the ejection fraction is approximately?

A

0.6

How well did you know this?
1
Not at all
2
3
4
5
Perfectly
16
Q

Sustained elevation of cardiac output will occur with which of the following conditions?

A

Anemia

How well did you know this?
1
Not at all
2
3
4
5
Perfectly
17
Q

A diastolic murmur accompanied by an increased preload can be caused by

A

Aortic regurgitation

How well did you know this?
1
Not at all
2
3
4
5
Perfectly
18
Q

Increasing vagal stimulation of the heart will cause an increase in

A

PR interval

How well did you know this?
1
Not at all
2
3
4
5
Perfectly
19
Q

During exercise, there is an increase in a person’s

A

Stroke volume

How well did you know this?
1
Not at all
2
3
4
5
Perfectly
20
Q

Phase-4 depolarization of SA nodal cells is caused by

A

An increase in the flow of sodium into the cell

How well did you know this?
1
Not at all
2
3
4
5
Perfectly
21
Q

Cardiovascular changes that occur during inspiration include decreased

A

Systemic blood pressure

How well did you know this?
1
Not at all
2
3
4
5
Perfectly
22
Q

Blood pressure increases and heart rate decreases in response to

A

Increased intracranial pressure

How well did you know this?
1
Not at all
2
3
4
5
Perfectly
23
Q

During exercise, cardiac output is augmented by

A

Increased ventricular contractility

How well did you know this?
1
Not at all
2
3
4
5
Perfectly
24
Q

When flow through the mitral valve is restricted by mitral stenosis,

A

Exercise can induce acute pulmonary edema

How well did you know this?
1
Not at all
2
3
4
5
Perfectly
25
Q

Stroke volume can be decreased by

A

Increasing heart rate

How well did you know this?
1
Not at all
2
3
4
5
Perfectly
26
Q

An ectopic extrasystole caused by a ventricular focus is characterized by

A

A skipped ventricular contraction

How well did you know this?
1
Not at all
2
3
4
5
Perfectly
27
Q

The electrocardiogram is most effective in detecting a decrease in

A

Coronary blood flow

How well did you know this?
1
Not at all
2
3
4
5
Perfectly
28
Q

Stroke volume can be increased by

A

Decreasing total peripheral resistance

How well did you know this?
1
Not at all
2
3
4
5
Perfectly
29
Q

Which one of the following will be observed in a patient with a dilated failing left ventricle and a normal resting mean blood pressure?

A

An increased pulmonary capillary hydrostatic pressure

How well did you know this?
1
Not at all
2
3
4
5
Perfectly
30
Q

Which one of the following correctly describes an event that normally occurs during the PR interval?

A

The cardiac action potential passes through the AV node

How well did you know this?
1
Not at all
2
3
4
5
Perfectly
31
Q

Left ventricular wall stress will be decreased by an increase in

A

The thickness of the free wall of the left ventricle

How well did you know this?
1
Not at all
2
3
4
5
Perfectly
32
Q

Which one of the following will be observed in a patient with aortic regurgitation?

A

An increased ejection fraction

How well did you know this?
1
Not at all
2
3
4
5
Perfectly
33
Q

At which point on the above ventricular action potential is membrane potential most dependent on calcium permeability?

A

Point D

How well did you know this?
1
Not at all
2
3
4
5
Perfectly
34
Q

During which interval on the ECG does the bundle of His depolarize?

A

PR

How well did you know this?
1
Not at all
2
3
4
5
Perfectly
35
Q

The diagnosis of a first-degree heart block is made if

A

The PR interval of the ECG is increased

How well did you know this?
1
Not at all
2
3
4
5
Perfectly
36
Q

Positive inotropic drugs such a digitalis reduce ischemic cardiac pain (angina) in a dilated failing heart by

A

Decreasing preload

How well did you know this?
1
Not at all
2
3
4
5
Perfectly
37
Q

The mitral valve opens at point

A

A

How well did you know this?
1
Not at all
2
3
4
5
Perfectly
38
Q

The second heart sound begins at point

A

B

How well did you know this?
1
Not at all
2
3
4
5
Perfectly
39
Q

Systole begins at point

A

E

How well did you know this?
1
Not at all
2
3
4
5
Perfectly
40
Q

A major sign of cardiac tamponade is

A

Pulsus paradoxus

How well did you know this?
1
Not at all
2
3
4
5
Perfectly
41
Q

A mild hemorrhage will cause stroke volume to shift from point X to point

A

A

How well did you know this?
1
Not at all
2
3
4
5
Perfectly
42
Q

An increase in afterload and venous compliance can cause stroke volume to change from the point marked X to point

A

C

How well did you know this?
1
Not at all
2
3
4
5
Perfectly
43
Q

The upstroke of the SA nodal action potential is produced by opening a channel that is

A

Primarily permeable to Na+

44
Q

The channel responsible for the initiation of phase-4 depolarization in SA nodal cells

A

Is primarily permeable to Na+

45
Q

Sympathetic stimulation of the heart results in

A

An increase in the activity of the SR calcium pump

46
Q

An exercise stress test to rule out ischemic heart disease is positive if

A

The ST segment of the ECG is depressed

47
Q

A 42-year-old woman with mitral prolapse is admitted to the hospital for evaluation of her cardiac function. Which of the following values is the best index of the preload on her heart?

A

Left ventricular end-diastolic volume

48
Q

A patient presents to the Emergency Department with intermittent chest pain. The ECG and blood tests are negative for myocardial infarction, but the echocardiogram shows thickening of the left ventricular muscle and narrowing of the aortic valve. Medications to lower afterload are prescribed. Which of the following values would provide the best measure of the effectiveness of the medication in lowering left ventricular afterload in this patient?

A

Left ventricular mean systolic pressure

49
Q

A 55-year-old male reports several recent episodes of syncope. An electrocardiogram is performed. Which of the following arrhythmias is most commonly associated with syncope?

A

Third-degree heart block

50
Q

The spouse of a 58-year-old-man calls 9-1-1 because her husband complains of chest pain radiating down his left arm. He is transported to the Emergency Department, where an electrocardiogram and cardiac enzymes indicate a recent myocardial infarction. The man is sent for a cardiac catheterization, including coronary angiography and hemodynamic recordings throughout the cardiac cycle. No valvular defects were present. During ventricular ejection, the pressure difference smallest in magnitude is between which of the following?

A

Left ventricle and aorta

51
Q

An 82-year-old woman was admitted to the hospital with ascites, peripheral edema, and shortness of breath. Cardiac catheterization was ordered and the following values were obtained:
Pulmonary artery O2 content = 20 mL/100 mL
Pulmonary vein O2 content = 12 mL/100 mL
Oxygen consumption = 280 mL/min
Stroke volume = 40 mL
What is the woman’s cardiac output?

A

3.5 L/min

52
Q

A 66-year-old male is referred to a cardiologist for evaluation. Physical examination reveals a diastolic murmur prominent over the left sternal border, a decrease in diastolic pressure, and an increase in pulse pressure. Which of the following is the most likely diagnosis?

A

Aortic regurgitation

53
Q

A patient undergoes cardiac transplantation for severe idiopathic cardiomyopathy. Upon release from the hospital, the patient is referred to a cardiac rehabilitation program. The exercise technologist starts the patient on a walking regimen. In transplant patients, stroke volume may increase during exercise by which of the following mechanisms?

A

A decrease in venous compliance

54
Q

A patient complaining of an irregular heart beat is referred for a cardiac electrophysiological (EP) study. Propagation of the action potential through the heart is fastest in which of the following?

A

Purkinje fibers

55
Q

A 75-year-old woman presents with fatigue, edema, and shortness of breath. Her physician prescribes a diuretic and a positive inotropic agent. Which of the following changes is primarily responsible for the improvement in her condition?

A

A reduction in heart size

56
Q

A 37-year-old woman undergoes a CT scan of the abdomen, which reveals a large peritoneal mass. A subsequent magnetic resonance angiography study showed that the abdominal aorta was constricted to one-half of its resting diameter. As a result, resistance to blood flow through the vessel would be which of the following?

A

Increased 16-fold

57
Q

A 52-year-old man presents with tar-black stools. An endoscopy is ordered and reveals a bleeding ulcer in the antrum of the stomach. Baroreflex-induced compensation for the mild hemorrhage will cause which of the following values to be lower than it was before the hemorrhage?

A

Venous capacitance

58
Q

A 22-year-old woman is hospitalized with a history of respiratory distress, fever, and fatigue. ST segment and T wave abnormalities suggest myocarditis, which is attributed to an acute viral origin. Over the next several days, significant peripheral edema develops. The edema is most likely caused by which of the following?

A

Increased central venous pressure

59
Q

A patient undergoes cardiac catheterization to assess his left ventricular function prior to thoracic surgery. What is his ejection fraction, as determined from the left ventricular pressure-volume curve illustrated below.

A

0.54

60
Q

A 38-year-old man has a murmur that ceases with the onset of the second heart sound. The second heart sound occurs at the onset of which phase of the cardiac cycle?

A

Isovolumetric relaxation

61
Q

A 58-year-old woman with idiopathic pulmonary hypertension presents with right ventricular hypertrophy and cor pulmonale. Her electrocardiogram shows positive QRS complexes in leads V1, III, and aVF, and equiphasic QRS complexes in lead aVR. Which of the following is her mean QRS

A

+120°

62
Q

A 57-year-old male complains of an irregular heart beat that he notices is relieved by pressing on his eyeball. An electrocardiogram reveals atrial fibrillation. Which of the following is most likely to accompany this condition?

A

An increased left atrial pressure

63
Q

A 24-year-old woman undergoes an annual physical examination for participation on the varsity track team at her college. While auscultating her heart sounds, the sports medicine physician instructs the woman to take in a deep inspiration. During this maneuver, he detects splitting of the second heart sound. Which of the following is the mechanism underlying this finding?

A

Delayed closing of the pulmonic valve

64
Q

A 55-year-old female presents for her annual physical examination. Upon auscultation, a third heart sound is heard. The differential diagnosis of this finding includes which of the following?

A

Anemia

65
Q

A 23-year-old female presents with fatigue and is found to have a systolic murmur and higher than normal cardiac output. The differential diagnosis based on these findings includes which of the following?

A

Anemia

66
Q

Ventricular pressure-volume curves are determined in two different patients, as illustrated below. Which of the following is greater in Patient #1?

A

Cardiac efficiency

67
Q

Physical examination of a 41-year-old narcotic abusor reveals an early systolic murmur. The physician also notes a 7-cm distance between the height of the blood in his right internal jugular vein and sternal angle (normal = 3 cm). Which of the following conditions is most likely responsible for the physical findings?

A

Tricuspid regurgitation

68
Q

A 42-year-old woman with lightheadedness and recurrent syncope is taken to the emergency department where she is given atropine. Her symptoms are relieved by an increase in which of the following?

A

Heart rate

69
Q

A 32-year-old female complains of intermittent chest discomfort that occurs most frequently when she drinks lots of coffee to stay up to meet deadlines at work. She is referred to cardiology for an exercise stress test to rule out cardiac ischemia as the cause for her angina. The test will be considered positive if which of the following occurs?

A

ST segment depression occurs

70
Q

A postlaminectomy patient suddenly stands up after being supine for 8 hours. As a result, which of the following hemodynamic variables will increase?

A

Ejection fraction

71
Q

A newborn baby is cyanotic upon delivery. The cyanosis is not relieved by breathing 100% oxygen. A diagnosis of persistent fetal circulation is made based on which of the following clinical signs?

A

Pulmonary vasoconstriction and hypertension

72
Q

A 19-year-old male severs an artery in a motorcycle accident. A bystander applies a tourniquet to stop the bleeding. When the paramedics arrive, the blood pressure of the injured man was only slightly hypotensive and his pupils were reactive. The greatest percentage of the redistributed blood volume came from which of the following?

A

Venules and veins

73
Q

An 84-year-old woman presents with paroxysmal dizziness, syncope, confusion, and fatigue. Her heart rate did not change when the patient was instructed to perform a Valsalva maneuver. 24-hour Holter monitoring revealed periodic episodes of sinus bradycardia. Phase-4 depolarization of SA nodal cells is caused by which of the following?

A

An increase in the flow of sodium into the cell

74
Q

During auscultation of a patient with long-standing hypertension, the physician notes that the splitting of the second heart sound is reversed with P2 occurring before A2. Which of the following is a common electrocardiographic finding accompanying paradoxical splitting of the second heart sound?

A

Left bundle branch block

75
Q

An 83-year-old woman with long-standing hypertension presents after a near-syncopal episode upon standing. Her blood pressure is taken sitting and then standing. Systolic pressure decreased slightly and pulse pressure increased in the standing position. Which of the following can lead to an increased pulse pressure?

A

A decrease in aortic compliance

76
Q

A 75-year-old woman makes an appointment to see her physician because she is having breathing difficulties when she lies down at night. An S3 gallop is heard upon auscultation. Her signs and symptoms are most likely due to which of the following?

A

Left heart failure

77
Q

A cardiac catheterization is performed on a 39-year-old man who presents with angina. The left ventricular pressure-volume curve shows a decreased stroke volume and ejection fraction. Which of the following mechanisms may compromise stroke volume following myocardial infarction?

A

An increase in heart rate

78
Q

A 42-year-old athlete becomes alarmed when he notices a series of heart palpitations several hours after he exercises. After examining the patient’s ECG, the physician notes a sinus rhythm with occasional PVCs (premature ventricular complexes). The physician tells his patient that the palpitations are due to interpolated beats and that they are not a cause for alarm. Benign PVCs may occur because the athlete has which of the following?

A

Bradycardia

79
Q

A 56-year-old male was admitted to the hospital with angina and diaphoresis. A myocardial infarction is suspected, and a 12-lead electrocardiogram (ECG) is ordered. The ECG is most effective in detecting a decrease in which of the following?

A

Coronary blood flow

80
Q

A 48-year-old sedentary, obese male with four-vessel coronary occlusive disease has a massive myocardial infarction while shoveling snow. In the blizzard conditions, it takes the ambulance over an hour to reach the man’s home. When the paramedics arrive, the patient’s radial pulse is rapid and thready, he has pink froth coming from his mouth, and he is nonresponsive. Increasing which of the following would lead to an increased stroke volume in cardiogenic shock?

A

Ventricular contractility

81
Q

A patient comes to his physician complaining that he is no longer able to exercise as long as he used to. The physician auscultates crepitant rales and a third heart sound; blood pressure is normal. He sends the patient to cardiology to rule out heart failure. Which of the following is most consistent with a diagnosis of left heart failure?

A

An increased left ventricular wall stress

82
Q

A pacemaker is inserted in a patient in order to shorten the PR interval detected on their ECG. Which of the following events normally occurs during the PR interval?

A

The cardiac action potential passes through the AV node

83
Q

A 47-year-old female is brought to the Emergency Department because she fainted at the gym during her daily aerobic workout. A prominent systolic murmur is heard and a presumptive diagnosis of aortic stenosis is made. Which of the following is consistent with that diagnosis?

A

A decreased pulse pressure

84
Q

A patient with an inferior MI develops a stable bradycardia of 50/min. The cardiologist orders an ECG to evaluate whether there is sinus node dysfunction or an atrioventricular conduction disturbance. The diagnosis of a first-degree heart block is made in which of the following cases?

A

The PR interval of the ECG is increased

85
Q

A 67-year-old man who has difficulty breathing when he exercises makes an appointment to see his physician. Auscultation reveals a holosystolic murmur leading to the diagnosis of mitral regurgitation. Which of the following laboratory findings is most likely to be present?

A

An increased a wave

86
Q

A 43-year-old male comes to his physician complaining of exhaustion and shortness of breath. After completing the physical exam, the physician suspects the patient may be suffering from cardiac tamponade. Which of the following observations led to the physician’s putative diagnosis?

A

Pulsus paradoxus

87
Q

An EMT arrives at the scene of an automobile accident, and finds a hemorrhaging, unconscious young woman. Which of the following is a sign of hemorrhagic shock?

A

Low hematocrit

88
Q

A 37-year-old patient is brought to the Emergency Department in shock. Which of the following is a reason to direct treatment toward anaphylactic shock rather than hypovolumic shock?

A

Cardiac output is higher than normal

89
Q

A woman with syndrome X is prescribed a low-calorie diet and 30-min of daily aerobic exercise. Sympathetic stimulation during exercise has which of the following cardiac effects?

A

An increase in the activity of the SR calcium pump

90
Q

A 23-year-old collegiate dance squad member adopts a sedentary lifestyle once she starts medical school. After the Gross Anatomy course, she decides to restore her state of physical fitness by resuming a regular exercise routine. The cardiovascular responses to isotonic exercise include an increase in which of the following?

A

Stroke volume

91
Q

A 2-year-old boy is mauled by a black bear while hiking with his family in the Appalachian mountains. A claw puncture wound to the skull compressed the underlying brain tissue. Which of the following occurs in response to an increased intracranial pressure?

A

Blood pressure increases and heart rate decreases

92
Q

Left ventricular wall stress will be decreased by an increase in which of the following?

A

The thickness of the free wall of the left ventricle

93
Q

A 48-year-old man develops chest pain while running. His wife takes him to the Emergency Department, where the following ECG is obtained. The electrocardiographic changes are consistent with a diagnosis of which of the following?

A

Anterior infarction

94
Q

A patient with Type II diabetes reports for his 6-month check-up. His doctor prescribes a daily 30-minute routine of walking at a brisk pace. During aerobic exercise, blood flow remains relatively constant within which of the following?

A

Brain

95
Q

A 56-year-old man presents with complaints of fatigue and headaches. During the physical examination, he is found to have a wide pulse pressure. Which of the following conditions causes pulse pressure to increase?

A

Hypertension

96
Q

A 22-year-old male ruptures his spleen in a motorcycle accident. A reduction in blood pressure would cause a decrease in which of the following?

A

Venous compliance

97
Q

Which of the following organs has the highest arteriovenous O2 difference under normal resting conditions?

A

Heart

98
Q

A 63-year-old woman with congestive heart failure is given digitalis. Positive inotropic drugs can reduce ischemic cardiac pain (angina) in a dilated failing heart by doing which of the following?

A

Decreasing preload

99
Q

Which of the following characteristics is most similar in the systemic and pulmonary circulations?

A

Preload

100
Q

A 6-year-old girl undergoes a routine physical examination for entry into the first grade. She is found to be tachycardic, and has a wide pulse pressure. A thrill and a continuous murmur with late systolic accentuation at the upper left sternal edge are detected upon auscultation. Echocardiography reveals a patent ductus arteriosus. Which of the following best describes the function of the ductus arteriosus in the fetal circulation?

A

It diverts oxygenated blood away from the lungs to the aorta

101
Q

A 63-year-old woman presented with acute onset of right eye pain. Ophthalmic and neurologic examinations were normal except for a loud right carotid bruit. The eye pain ceased following carotid endarterectomy. The bruit was most likely caused by which of the following?

A

A high velocity of blood within the carotid artery

102
Q

A 57-year-old woman is undergoing a femoral popliteal bypass for her peripheral vascular disease. The vascular surgeon wishes to induce a localized arteriolar constriction to help control hemostasis. An increase in the local concentration of which of the following agents will cause systemic vasoconstriction?

A

Angiotensin II

103
Q

Which of the following is true regarding the functional closure of the ductus arteriosus?

A

It is the final event required for conversion of the transitional circulation in the newborn to the adult circulatory pattern

104
Q

At which of the following sites in the cardiovascular system does the blood flow lose the greatest amount of energy?

A

Arterioles

105
Q

During the cardiac cycle, when does the highest coronary blood flow per gram of left ventricular myocardium occur.

A

At the beginning of diastole

106
Q

A 59-year-old male with an ejection fraction of 15%,who is being treated with medications for his heart failure, is asked whether he would like to participate in a trial for an experimental drug. The drug being tested is designed to decrease the expression of phospholamban on ventricular muscle cells. Which of the following would be increased by decreasing phospholamban?

A

The concentration of calcium within the SR

107
Q

A 29-year-old woman presents at the obstetrician/gynecologist’s office with breast tenderness. She’s concerned she may have a lump because her mother had breast cancer, and reports that she can’t be pregnant because she just finished her menstrual period about 2 weeks ago. An assay for human chorionic gonadotropin (hCG) in her urine is positive, confirming pregnancy. In the developing fetus, which vessel has the greatest partial pressure of oxygen?

A

Umbilical vein